當前位置: 華文星空 > 知識

如何計算這個極限?

2021-03-06知識

\lim_{n \rightarrow \infty}{\frac{\ln^2{n}}{n}\sum_{k=2}^{n-2}{\frac{1}{\ln k\ln(n-k)}}}

@23Square 給出的答案已然完備。

其中一個方向的放縮比較簡單:

{\frac{\ln^2{n}}{n}\sum_{k=2}^{n-2}{\frac{1}{\ln k\ln(n-k)}}}\geq1-\frac{3}{n}

在這裏給出另一個方向的放縮的一種思路:

由Cauchy-Schwarz不等式

{\frac{\ln^2{n}}{n}\sum_{k=2}^{n-2}{\frac{1}{\ln k\ln(n-k)}}}\leq{\frac{\ln^2{n}}{n}\sum_{k=2}^{n-2}{\frac{1}{\ln^2{k}}}}

再由O’Stolz定理及迫斂性準則,易得

\lim_{n \to \infty} {\frac{\ln^2{n}}{n}\sum_{k=2}^{n-2}{\frac{1}{\ln^2{k}}}}=1

由迫斂性準則,所求極限為1.